Why $ lim_{nrightarrow infty} frac{n!}{n^{k}(n-k)! } =1 $?












2












$begingroup$


I was on brilliant.org learning probability. There was a process explaining how the distribution of a Poisson Random Variable can be obtained from a Binomial Random Variable.



Consider the binomial distribution:



$$
begin{equation}begin{aligned}
P(X=k) &={binom n k} p^k (1-p)^{n-k}\
&=frac{n!}{k!(n-k)!} p^k (1-p)^{n-k}
end{aligned}end{equation}
$$


Substitute $m=np$ , or $p=frac{m}{n}$ :
$$
begin{equation}begin{aligned}
P(X=k) &=frac{n!}{k!(n-k)!} left(frac{m}{n}right)^k left(1-frac{m}{n}right)^{n-k}\
&=frac{n!}{k!(n-k)!} frac{m^k}{n^k} left(1-frac{m}{n}right)^{n}left(1-frac{m}{n}right)^{-k}
end{aligned}end{equation}
$$


Slightly rearrange
$$
begin{equation}begin{aligned}
&=frac{n!}{n^k(n-k)!} left(1-frac{m}{n}right)^{-k}frac{m^k}{k!}left(1-frac{m}{n}right)^{n}
end{aligned}end{equation}
$$




Note that
$$
begin{equation}begin{aligned}
& lim_{nrightarrow infty} frac{n!}{n^{k}(n-k)! } =1,quadlim_{nrightarrow infty} left(1-frac{m}{n}right)^{-k} =1,quad lim_{nrightarrow infty} left(1-frac{m}{n}right)^{n} =e^{-m}
end{aligned}end{equation}
$$




Thus, we have the final result which is equal to the formula for the Poisson distribution.



$$
=frac{m^k e^{-m}}{k!}
$$




In all these steps, what I don't understand is the following limit:
$$
lim_{nrightarrow infty} frac{n!}{n^{k}(n-k)! } =1
$$











share|cite|improve this question











$endgroup$












  • $begingroup$
    There are several posts about this: Why does $lim_{ntoinfty} frac{n!}{(n-k)!n^k}$ equal 1, Finding limit of sequence: $lim _{n to infty} {frac{n!}{n^k(n-k)!}}=1$, Proof that $limlimits_{h to infty} frac{h!}{h^k(h-k)!}=1$ for any $k$, Limits involing Factorials $lim_{Ntoinfty} frac{N!}{(N-k)!N^{k}}$
    $endgroup$
    – Martin Sleziak
    yesterday












  • $begingroup$
    I found the posts in the above comment using Approach0. For some useful tips on searching here see: How to search on this site?
    $endgroup$
    – Martin Sleziak
    yesterday










  • $begingroup$
    Some of the other posts treating the same question painfully lack details and context. Maybe you'd want to put them on hold or close them.
    $endgroup$
    – billyandr
    yesterday












  • $begingroup$
    billyandr: If you actually have a look at those links, you can see that two of those posts are closed (as duplicates) now. Let me also say that the fact that you have added some more context to your question is certainly appreciated. (After all, that's what lead to reopening.)
    $endgroup$
    – Martin Sleziak
    yesterday
















2












$begingroup$


I was on brilliant.org learning probability. There was a process explaining how the distribution of a Poisson Random Variable can be obtained from a Binomial Random Variable.



Consider the binomial distribution:



$$
begin{equation}begin{aligned}
P(X=k) &={binom n k} p^k (1-p)^{n-k}\
&=frac{n!}{k!(n-k)!} p^k (1-p)^{n-k}
end{aligned}end{equation}
$$


Substitute $m=np$ , or $p=frac{m}{n}$ :
$$
begin{equation}begin{aligned}
P(X=k) &=frac{n!}{k!(n-k)!} left(frac{m}{n}right)^k left(1-frac{m}{n}right)^{n-k}\
&=frac{n!}{k!(n-k)!} frac{m^k}{n^k} left(1-frac{m}{n}right)^{n}left(1-frac{m}{n}right)^{-k}
end{aligned}end{equation}
$$


Slightly rearrange
$$
begin{equation}begin{aligned}
&=frac{n!}{n^k(n-k)!} left(1-frac{m}{n}right)^{-k}frac{m^k}{k!}left(1-frac{m}{n}right)^{n}
end{aligned}end{equation}
$$




Note that
$$
begin{equation}begin{aligned}
& lim_{nrightarrow infty} frac{n!}{n^{k}(n-k)! } =1,quadlim_{nrightarrow infty} left(1-frac{m}{n}right)^{-k} =1,quad lim_{nrightarrow infty} left(1-frac{m}{n}right)^{n} =e^{-m}
end{aligned}end{equation}
$$




Thus, we have the final result which is equal to the formula for the Poisson distribution.



$$
=frac{m^k e^{-m}}{k!}
$$




In all these steps, what I don't understand is the following limit:
$$
lim_{nrightarrow infty} frac{n!}{n^{k}(n-k)! } =1
$$











share|cite|improve this question











$endgroup$












  • $begingroup$
    There are several posts about this: Why does $lim_{ntoinfty} frac{n!}{(n-k)!n^k}$ equal 1, Finding limit of sequence: $lim _{n to infty} {frac{n!}{n^k(n-k)!}}=1$, Proof that $limlimits_{h to infty} frac{h!}{h^k(h-k)!}=1$ for any $k$, Limits involing Factorials $lim_{Ntoinfty} frac{N!}{(N-k)!N^{k}}$
    $endgroup$
    – Martin Sleziak
    yesterday












  • $begingroup$
    I found the posts in the above comment using Approach0. For some useful tips on searching here see: How to search on this site?
    $endgroup$
    – Martin Sleziak
    yesterday










  • $begingroup$
    Some of the other posts treating the same question painfully lack details and context. Maybe you'd want to put them on hold or close them.
    $endgroup$
    – billyandr
    yesterday












  • $begingroup$
    billyandr: If you actually have a look at those links, you can see that two of those posts are closed (as duplicates) now. Let me also say that the fact that you have added some more context to your question is certainly appreciated. (After all, that's what lead to reopening.)
    $endgroup$
    – Martin Sleziak
    yesterday














2












2








2


1



$begingroup$


I was on brilliant.org learning probability. There was a process explaining how the distribution of a Poisson Random Variable can be obtained from a Binomial Random Variable.



Consider the binomial distribution:



$$
begin{equation}begin{aligned}
P(X=k) &={binom n k} p^k (1-p)^{n-k}\
&=frac{n!}{k!(n-k)!} p^k (1-p)^{n-k}
end{aligned}end{equation}
$$


Substitute $m=np$ , or $p=frac{m}{n}$ :
$$
begin{equation}begin{aligned}
P(X=k) &=frac{n!}{k!(n-k)!} left(frac{m}{n}right)^k left(1-frac{m}{n}right)^{n-k}\
&=frac{n!}{k!(n-k)!} frac{m^k}{n^k} left(1-frac{m}{n}right)^{n}left(1-frac{m}{n}right)^{-k}
end{aligned}end{equation}
$$


Slightly rearrange
$$
begin{equation}begin{aligned}
&=frac{n!}{n^k(n-k)!} left(1-frac{m}{n}right)^{-k}frac{m^k}{k!}left(1-frac{m}{n}right)^{n}
end{aligned}end{equation}
$$




Note that
$$
begin{equation}begin{aligned}
& lim_{nrightarrow infty} frac{n!}{n^{k}(n-k)! } =1,quadlim_{nrightarrow infty} left(1-frac{m}{n}right)^{-k} =1,quad lim_{nrightarrow infty} left(1-frac{m}{n}right)^{n} =e^{-m}
end{aligned}end{equation}
$$




Thus, we have the final result which is equal to the formula for the Poisson distribution.



$$
=frac{m^k e^{-m}}{k!}
$$




In all these steps, what I don't understand is the following limit:
$$
lim_{nrightarrow infty} frac{n!}{n^{k}(n-k)! } =1
$$











share|cite|improve this question











$endgroup$




I was on brilliant.org learning probability. There was a process explaining how the distribution of a Poisson Random Variable can be obtained from a Binomial Random Variable.



Consider the binomial distribution:



$$
begin{equation}begin{aligned}
P(X=k) &={binom n k} p^k (1-p)^{n-k}\
&=frac{n!}{k!(n-k)!} p^k (1-p)^{n-k}
end{aligned}end{equation}
$$


Substitute $m=np$ , or $p=frac{m}{n}$ :
$$
begin{equation}begin{aligned}
P(X=k) &=frac{n!}{k!(n-k)!} left(frac{m}{n}right)^k left(1-frac{m}{n}right)^{n-k}\
&=frac{n!}{k!(n-k)!} frac{m^k}{n^k} left(1-frac{m}{n}right)^{n}left(1-frac{m}{n}right)^{-k}
end{aligned}end{equation}
$$


Slightly rearrange
$$
begin{equation}begin{aligned}
&=frac{n!}{n^k(n-k)!} left(1-frac{m}{n}right)^{-k}frac{m^k}{k!}left(1-frac{m}{n}right)^{n}
end{aligned}end{equation}
$$




Note that
$$
begin{equation}begin{aligned}
& lim_{nrightarrow infty} frac{n!}{n^{k}(n-k)! } =1,quadlim_{nrightarrow infty} left(1-frac{m}{n}right)^{-k} =1,quad lim_{nrightarrow infty} left(1-frac{m}{n}right)^{n} =e^{-m}
end{aligned}end{equation}
$$




Thus, we have the final result which is equal to the formula for the Poisson distribution.



$$
=frac{m^k e^{-m}}{k!}
$$




In all these steps, what I don't understand is the following limit:
$$
lim_{nrightarrow infty} frac{n!}{n^{k}(n-k)! } =1
$$








limits factorial






share|cite|improve this question















share|cite|improve this question













share|cite|improve this question




share|cite|improve this question








edited yesterday







billyandr

















asked yesterday









billyandrbillyandr

237




237












  • $begingroup$
    There are several posts about this: Why does $lim_{ntoinfty} frac{n!}{(n-k)!n^k}$ equal 1, Finding limit of sequence: $lim _{n to infty} {frac{n!}{n^k(n-k)!}}=1$, Proof that $limlimits_{h to infty} frac{h!}{h^k(h-k)!}=1$ for any $k$, Limits involing Factorials $lim_{Ntoinfty} frac{N!}{(N-k)!N^{k}}$
    $endgroup$
    – Martin Sleziak
    yesterday












  • $begingroup$
    I found the posts in the above comment using Approach0. For some useful tips on searching here see: How to search on this site?
    $endgroup$
    – Martin Sleziak
    yesterday










  • $begingroup$
    Some of the other posts treating the same question painfully lack details and context. Maybe you'd want to put them on hold or close them.
    $endgroup$
    – billyandr
    yesterday












  • $begingroup$
    billyandr: If you actually have a look at those links, you can see that two of those posts are closed (as duplicates) now. Let me also say that the fact that you have added some more context to your question is certainly appreciated. (After all, that's what lead to reopening.)
    $endgroup$
    – Martin Sleziak
    yesterday


















  • $begingroup$
    There are several posts about this: Why does $lim_{ntoinfty} frac{n!}{(n-k)!n^k}$ equal 1, Finding limit of sequence: $lim _{n to infty} {frac{n!}{n^k(n-k)!}}=1$, Proof that $limlimits_{h to infty} frac{h!}{h^k(h-k)!}=1$ for any $k$, Limits involing Factorials $lim_{Ntoinfty} frac{N!}{(N-k)!N^{k}}$
    $endgroup$
    – Martin Sleziak
    yesterday












  • $begingroup$
    I found the posts in the above comment using Approach0. For some useful tips on searching here see: How to search on this site?
    $endgroup$
    – Martin Sleziak
    yesterday










  • $begingroup$
    Some of the other posts treating the same question painfully lack details and context. Maybe you'd want to put them on hold or close them.
    $endgroup$
    – billyandr
    yesterday












  • $begingroup$
    billyandr: If you actually have a look at those links, you can see that two of those posts are closed (as duplicates) now. Let me also say that the fact that you have added some more context to your question is certainly appreciated. (After all, that's what lead to reopening.)
    $endgroup$
    – Martin Sleziak
    yesterday
















$begingroup$
There are several posts about this: Why does $lim_{ntoinfty} frac{n!}{(n-k)!n^k}$ equal 1, Finding limit of sequence: $lim _{n to infty} {frac{n!}{n^k(n-k)!}}=1$, Proof that $limlimits_{h to infty} frac{h!}{h^k(h-k)!}=1$ for any $k$, Limits involing Factorials $lim_{Ntoinfty} frac{N!}{(N-k)!N^{k}}$
$endgroup$
– Martin Sleziak
yesterday






$begingroup$
There are several posts about this: Why does $lim_{ntoinfty} frac{n!}{(n-k)!n^k}$ equal 1, Finding limit of sequence: $lim _{n to infty} {frac{n!}{n^k(n-k)!}}=1$, Proof that $limlimits_{h to infty} frac{h!}{h^k(h-k)!}=1$ for any $k$, Limits involing Factorials $lim_{Ntoinfty} frac{N!}{(N-k)!N^{k}}$
$endgroup$
– Martin Sleziak
yesterday














$begingroup$
I found the posts in the above comment using Approach0. For some useful tips on searching here see: How to search on this site?
$endgroup$
– Martin Sleziak
yesterday




$begingroup$
I found the posts in the above comment using Approach0. For some useful tips on searching here see: How to search on this site?
$endgroup$
– Martin Sleziak
yesterday












$begingroup$
Some of the other posts treating the same question painfully lack details and context. Maybe you'd want to put them on hold or close them.
$endgroup$
– billyandr
yesterday






$begingroup$
Some of the other posts treating the same question painfully lack details and context. Maybe you'd want to put them on hold or close them.
$endgroup$
– billyandr
yesterday














$begingroup$
billyandr: If you actually have a look at those links, you can see that two of those posts are closed (as duplicates) now. Let me also say that the fact that you have added some more context to your question is certainly appreciated. (After all, that's what lead to reopening.)
$endgroup$
– Martin Sleziak
yesterday




$begingroup$
billyandr: If you actually have a look at those links, you can see that two of those posts are closed (as duplicates) now. Let me also say that the fact that you have added some more context to your question is certainly appreciated. (After all, that's what lead to reopening.)
$endgroup$
– Martin Sleziak
yesterday










2 Answers
2






active

oldest

votes


















5












$begingroup$

It is rather obvious if you cancel the factorials:



$$frac{n!}{n^{k}(n-k)! } =frac{overbrace{n(n-1)cdots (n-k+1)}^{k; factors}}{n^k}= 1cdot left(1-frac{1}{n}right)cdots left(1-frac{k-1}{n}right)stackrel{n to infty}{longrightarrow} 1$$






share|cite|improve this answer









$endgroup$













  • $begingroup$
    Thank you so much. I didn't know it was right there under my eyes.
    $endgroup$
    – billyandr
    yesterday










  • $begingroup$
    You are welcome. This "not seeing the obvious" just happens once in a while, I think, to all who do maths. So, it is good to have a math platform like this one. :-)
    $endgroup$
    – trancelocation
    yesterday





















2












$begingroup$

$$a_n=frac{n!}{n^{k}(n-k)! }implies log(a_n)=log(n!)-k log(n)-log((n-k)!)$$



Use Stirling approximation and continue with Taylor series to get
$$log(a_n)=frac{k(1-k)}{2 n}+Oleft(frac{1}{n^2}right)$$ Continue with Taylor
$$a_n=e^{log(a_n)}=1+frac{k(1-k)}{2 n}+Oleft(frac{1}{n^2}right)$$






share|cite|improve this answer









$endgroup$









  • 1




    $begingroup$
    This has already a slight touch of overkill, hasn't it? :-)
    $endgroup$
    – trancelocation
    yesterday










  • $begingroup$
    @trancelocation. You are totally right for the limit. One of my manias is to always look at the approach to the limit. Have a look at matheducators.stackexchange.com/questions/8339/… . Cheers :-)
    $endgroup$
    – Claude Leibovici
    yesterday














Your Answer








StackExchange.ready(function() {
var channelOptions = {
tags: "".split(" "),
id: "69"
};
initTagRenderer("".split(" "), "".split(" "), channelOptions);

StackExchange.using("externalEditor", function() {
// Have to fire editor after snippets, if snippets enabled
if (StackExchange.settings.snippets.snippetsEnabled) {
StackExchange.using("snippets", function() {
createEditor();
});
}
else {
createEditor();
}
});

function createEditor() {
StackExchange.prepareEditor({
heartbeatType: 'answer',
autoActivateHeartbeat: false,
convertImagesToLinks: true,
noModals: true,
showLowRepImageUploadWarning: true,
reputationToPostImages: 10,
bindNavPrevention: true,
postfix: "",
imageUploader: {
brandingHtml: "Powered by u003ca class="icon-imgur-white" href="https://imgur.com/"u003eu003c/au003e",
contentPolicyHtml: "User contributions licensed under u003ca href="https://creativecommons.org/licenses/by-sa/3.0/"u003ecc by-sa 3.0 with attribution requiredu003c/au003e u003ca href="https://stackoverflow.com/legal/content-policy"u003e(content policy)u003c/au003e",
allowUrls: true
},
noCode: true, onDemand: true,
discardSelector: ".discard-answer"
,immediatelyShowMarkdownHelp:true
});


}
});














draft saved

draft discarded


















StackExchange.ready(
function () {
StackExchange.openid.initPostLogin('.new-post-login', 'https%3a%2f%2fmath.stackexchange.com%2fquestions%2f3185830%2fwhy-lim-n-rightarrow-infty-fracnnkn-k-1%23new-answer', 'question_page');
}
);

Post as a guest















Required, but never shown

























2 Answers
2






active

oldest

votes








2 Answers
2






active

oldest

votes









active

oldest

votes






active

oldest

votes









5












$begingroup$

It is rather obvious if you cancel the factorials:



$$frac{n!}{n^{k}(n-k)! } =frac{overbrace{n(n-1)cdots (n-k+1)}^{k; factors}}{n^k}= 1cdot left(1-frac{1}{n}right)cdots left(1-frac{k-1}{n}right)stackrel{n to infty}{longrightarrow} 1$$






share|cite|improve this answer









$endgroup$













  • $begingroup$
    Thank you so much. I didn't know it was right there under my eyes.
    $endgroup$
    – billyandr
    yesterday










  • $begingroup$
    You are welcome. This "not seeing the obvious" just happens once in a while, I think, to all who do maths. So, it is good to have a math platform like this one. :-)
    $endgroup$
    – trancelocation
    yesterday


















5












$begingroup$

It is rather obvious if you cancel the factorials:



$$frac{n!}{n^{k}(n-k)! } =frac{overbrace{n(n-1)cdots (n-k+1)}^{k; factors}}{n^k}= 1cdot left(1-frac{1}{n}right)cdots left(1-frac{k-1}{n}right)stackrel{n to infty}{longrightarrow} 1$$






share|cite|improve this answer









$endgroup$













  • $begingroup$
    Thank you so much. I didn't know it was right there under my eyes.
    $endgroup$
    – billyandr
    yesterday










  • $begingroup$
    You are welcome. This "not seeing the obvious" just happens once in a while, I think, to all who do maths. So, it is good to have a math platform like this one. :-)
    $endgroup$
    – trancelocation
    yesterday
















5












5








5





$begingroup$

It is rather obvious if you cancel the factorials:



$$frac{n!}{n^{k}(n-k)! } =frac{overbrace{n(n-1)cdots (n-k+1)}^{k; factors}}{n^k}= 1cdot left(1-frac{1}{n}right)cdots left(1-frac{k-1}{n}right)stackrel{n to infty}{longrightarrow} 1$$






share|cite|improve this answer









$endgroup$



It is rather obvious if you cancel the factorials:



$$frac{n!}{n^{k}(n-k)! } =frac{overbrace{n(n-1)cdots (n-k+1)}^{k; factors}}{n^k}= 1cdot left(1-frac{1}{n}right)cdots left(1-frac{k-1}{n}right)stackrel{n to infty}{longrightarrow} 1$$







share|cite|improve this answer












share|cite|improve this answer



share|cite|improve this answer










answered yesterday









trancelocationtrancelocation

14.1k1829




14.1k1829












  • $begingroup$
    Thank you so much. I didn't know it was right there under my eyes.
    $endgroup$
    – billyandr
    yesterday










  • $begingroup$
    You are welcome. This "not seeing the obvious" just happens once in a while, I think, to all who do maths. So, it is good to have a math platform like this one. :-)
    $endgroup$
    – trancelocation
    yesterday




















  • $begingroup$
    Thank you so much. I didn't know it was right there under my eyes.
    $endgroup$
    – billyandr
    yesterday










  • $begingroup$
    You are welcome. This "not seeing the obvious" just happens once in a while, I think, to all who do maths. So, it is good to have a math platform like this one. :-)
    $endgroup$
    – trancelocation
    yesterday


















$begingroup$
Thank you so much. I didn't know it was right there under my eyes.
$endgroup$
– billyandr
yesterday




$begingroup$
Thank you so much. I didn't know it was right there under my eyes.
$endgroup$
– billyandr
yesterday












$begingroup$
You are welcome. This "not seeing the obvious" just happens once in a while, I think, to all who do maths. So, it is good to have a math platform like this one. :-)
$endgroup$
– trancelocation
yesterday






$begingroup$
You are welcome. This "not seeing the obvious" just happens once in a while, I think, to all who do maths. So, it is good to have a math platform like this one. :-)
$endgroup$
– trancelocation
yesterday













2












$begingroup$

$$a_n=frac{n!}{n^{k}(n-k)! }implies log(a_n)=log(n!)-k log(n)-log((n-k)!)$$



Use Stirling approximation and continue with Taylor series to get
$$log(a_n)=frac{k(1-k)}{2 n}+Oleft(frac{1}{n^2}right)$$ Continue with Taylor
$$a_n=e^{log(a_n)}=1+frac{k(1-k)}{2 n}+Oleft(frac{1}{n^2}right)$$






share|cite|improve this answer









$endgroup$









  • 1




    $begingroup$
    This has already a slight touch of overkill, hasn't it? :-)
    $endgroup$
    – trancelocation
    yesterday










  • $begingroup$
    @trancelocation. You are totally right for the limit. One of my manias is to always look at the approach to the limit. Have a look at matheducators.stackexchange.com/questions/8339/… . Cheers :-)
    $endgroup$
    – Claude Leibovici
    yesterday


















2












$begingroup$

$$a_n=frac{n!}{n^{k}(n-k)! }implies log(a_n)=log(n!)-k log(n)-log((n-k)!)$$



Use Stirling approximation and continue with Taylor series to get
$$log(a_n)=frac{k(1-k)}{2 n}+Oleft(frac{1}{n^2}right)$$ Continue with Taylor
$$a_n=e^{log(a_n)}=1+frac{k(1-k)}{2 n}+Oleft(frac{1}{n^2}right)$$






share|cite|improve this answer









$endgroup$









  • 1




    $begingroup$
    This has already a slight touch of overkill, hasn't it? :-)
    $endgroup$
    – trancelocation
    yesterday










  • $begingroup$
    @trancelocation. You are totally right for the limit. One of my manias is to always look at the approach to the limit. Have a look at matheducators.stackexchange.com/questions/8339/… . Cheers :-)
    $endgroup$
    – Claude Leibovici
    yesterday
















2












2








2





$begingroup$

$$a_n=frac{n!}{n^{k}(n-k)! }implies log(a_n)=log(n!)-k log(n)-log((n-k)!)$$



Use Stirling approximation and continue with Taylor series to get
$$log(a_n)=frac{k(1-k)}{2 n}+Oleft(frac{1}{n^2}right)$$ Continue with Taylor
$$a_n=e^{log(a_n)}=1+frac{k(1-k)}{2 n}+Oleft(frac{1}{n^2}right)$$






share|cite|improve this answer









$endgroup$



$$a_n=frac{n!}{n^{k}(n-k)! }implies log(a_n)=log(n!)-k log(n)-log((n-k)!)$$



Use Stirling approximation and continue with Taylor series to get
$$log(a_n)=frac{k(1-k)}{2 n}+Oleft(frac{1}{n^2}right)$$ Continue with Taylor
$$a_n=e^{log(a_n)}=1+frac{k(1-k)}{2 n}+Oleft(frac{1}{n^2}right)$$







share|cite|improve this answer












share|cite|improve this answer



share|cite|improve this answer










answered yesterday









Claude LeiboviciClaude Leibovici

126k1158135




126k1158135








  • 1




    $begingroup$
    This has already a slight touch of overkill, hasn't it? :-)
    $endgroup$
    – trancelocation
    yesterday










  • $begingroup$
    @trancelocation. You are totally right for the limit. One of my manias is to always look at the approach to the limit. Have a look at matheducators.stackexchange.com/questions/8339/… . Cheers :-)
    $endgroup$
    – Claude Leibovici
    yesterday
















  • 1




    $begingroup$
    This has already a slight touch of overkill, hasn't it? :-)
    $endgroup$
    – trancelocation
    yesterday










  • $begingroup$
    @trancelocation. You are totally right for the limit. One of my manias is to always look at the approach to the limit. Have a look at matheducators.stackexchange.com/questions/8339/… . Cheers :-)
    $endgroup$
    – Claude Leibovici
    yesterday










1




1




$begingroup$
This has already a slight touch of overkill, hasn't it? :-)
$endgroup$
– trancelocation
yesterday




$begingroup$
This has already a slight touch of overkill, hasn't it? :-)
$endgroup$
– trancelocation
yesterday












$begingroup$
@trancelocation. You are totally right for the limit. One of my manias is to always look at the approach to the limit. Have a look at matheducators.stackexchange.com/questions/8339/… . Cheers :-)
$endgroup$
– Claude Leibovici
yesterday






$begingroup$
@trancelocation. You are totally right for the limit. One of my manias is to always look at the approach to the limit. Have a look at matheducators.stackexchange.com/questions/8339/… . Cheers :-)
$endgroup$
– Claude Leibovici
yesterday




















draft saved

draft discarded




















































Thanks for contributing an answer to Mathematics Stack Exchange!


  • Please be sure to answer the question. Provide details and share your research!

But avoid



  • Asking for help, clarification, or responding to other answers.

  • Making statements based on opinion; back them up with references or personal experience.


Use MathJax to format equations. MathJax reference.


To learn more, see our tips on writing great answers.




draft saved


draft discarded














StackExchange.ready(
function () {
StackExchange.openid.initPostLogin('.new-post-login', 'https%3a%2f%2fmath.stackexchange.com%2fquestions%2f3185830%2fwhy-lim-n-rightarrow-infty-fracnnkn-k-1%23new-answer', 'question_page');
}
);

Post as a guest















Required, but never shown





















































Required, but never shown














Required, but never shown












Required, but never shown







Required, but never shown

































Required, but never shown














Required, but never shown












Required, but never shown







Required, but never shown







Popular posts from this blog

How did Captain America manage to do this?

迪纳利

南乌拉尔铁路局